Find the mean, median, and mode of the following data. If necessary, round to one more decimal place than the largest number of decimal placesgiven in the dataRate of Fatal Alcohol ImpairedCar Crashes per 100 MillionVehicle Miles of Travel0.29 0.45 0.69 0.53 0.600.37 0.59 0.43 0.61 0.300.54 0.43 0.70 0.40 0.760.38 0.72 0.43 0.60 0.73Copy DataPrevAnswer 2 PointsKeypadKeyboard ShortcutsSeparate multiple answers with commas, if necessary.Selecting a button will replace the entered answer value(s) with the button value. If the button is not selected, the entered answer is used.Mean:Median:Mode:O No mode

Find The Mean, Median, And Mode Of The Following Data. If Necessary, Round To One More Decimal Place

Answers

Answer 1

The mean of the given data set is 0.528, the median of the given data set is 0.535 and the mode of the given data set is 0.43.

The given data set is:

0.29, 0.45, 0.69, 0.53, 0.60, 0.37, 0.59, 0.43, 0.61, 0.30, 0.54, 0.43, 0.70, 0.40, 0.76, 0.38, 0.72, 0.43, 0.60, 0.73.

Arranging the given data set in ascending order:

0.29, 0.30, 0.37, 0.38, 0.40, 0.43, 0.43, 0.43, 0.45, 0.53, 0.54, 0.59, 0.60, 0.60, 0.61, 0.69, 0.70, 0.72, 0.73, 0.76.

The mean of the data is given by;

Mean = (Sum of the numbers)/(total numbers)

=(0.29 + 0.30 + 0.37 + 0.38 + 0.40 + 0.43 + 0.43 + 0.43 + 0.45 + 0.53 + 0.54 + 0.59 + 0.60 + 0.60 + 0.61 + 0.69 + 0.70 + 0.72 + 0.73 + 0.76) / 20

= 10.55 / 20

= 0.5275 ≈ 0.528

Median = (0.53 + 0.54) / 2

             = 0.535

Mode = 0.43 (0.43 repeated most time)

Thus, the mean of the given data set is 0.528, the median of the given data set is 0.535 and the mode of the given data set is 0.43.

To learn more about mean, median, and mode visit:

https://brainly.com/question/15323584

#SPJ9


Related Questions

what is x? how would i find the value of x?

Answers

Given:

Find-:

The value of "x."

Explanation-:

Use a trigonometric is:

[tex]\tan\theta=\frac{Perpendicular}{\text{ Base}}[/tex]

In a triangle:

[tex]\begin{gathered} \text{ Angle}=x \\ \\ \text{ Base }=3 \\ \\ \text{ Pespendicular }=4 \end{gathered}[/tex]

The value of "x" is:

[tex]\begin{gathered} \tan\theta=\frac{\text{ Perpendicular}}{\text{ Base}} \\ \\ \tan x=\frac{4}{3} \\ \\ x=\tan^{-1}(\frac{4}{3}) \\ \\ x=53.13 \end{gathered}[/tex]

So, the angle is 53 degree

James bought a movie ticket for $4.05. Hepaid the movie ticket with quarters anddimes. If James used 18 coins in all, howmany quarters (q) and dimes (d) did he use?=q + d = 180.25q + 0.1d = 4.05+=ritorddia

Answers

q + d = 18

0.25q + 0.1d = 4.05

We will use substitution

Solving the first equation for q

q = 18 -d

Substituting this into the second equation

.25(18-d) + .1d = 4.05

Distribute

4.5 - .25d +.1d = 4.05

Combine like terms

4.5 - .15d = 4.05

Subtract 4.5 from each side

-.15d = -.45

Divide each side by -/15

-.15d/-.15 = -.45/-.15

d = 3

We have 3 dimes

Now we can find the number of quarters

q = 18-d

q = 18-3

q = 15

We have 15 quarters

all of the Patron in part shade and rewrite (x × y)^nas a product of two single powers

Answers

we have

(2*3)^5

we know that

(2*3)^5=(2^5)(3^5)

Rewrite each term as product of two single powers

so

(2^5)(3^5)=(2^3)(2^2)(3^3)(3^2)

Part c

we have

10^2/10^0

when divide, subtract the exponents

so

10^(2-0)

10^2

another way

Any number elevated to zero is equal to 1

so

10^0=1

substitute

10^2/1=10^2

Part f

we have

(2/3)^5

we know that

(2/3)^5=2^5/3^5

Solve the system by substitution. (If there is no solution, enter NO SOLUTION. If there are an infinite number of solutions, enter the general solution in terms of x, where x is any real number.)x − 0.2y = 2−10x + 2y = 10(x, y) =

Answers

[tex]\begin{cases}x-0.2y=2 \\ -10x+2y=10\end{cases}[/tex]

Start multiplying the first equation for 10

[tex]\begin{gathered} 10(x-0.2y=2) \\ 10x-2y=20 \end{gathered}[/tex]

add the resulting equation with the second equation

[tex]\begin{gathered} 10x-2y+(-10x+2y)=20+10 \\ 0=30\rightarrow false\text{ }0\ne30 \\ \end{gathered}[/tex]

Answer:

There is no solution for the system

log 2-log 5 can also be written as ?.

Answers

The formula for difference of two logarthimic terms are,

[tex]\log a-\log b=\log (\frac{a}{b})[/tex]

Determine the expression for log 2 -log 5.

[tex]\log 2-\log 5=\log (\frac{2}{5})[/tex]

Answer: log(2/5)

The polynomial, 3x2 - x +1, can be classified as a 2nd degree trinomial.True or false?

Answers

The polynomial given is:

[tex]3x^2\text{ - x + 1}[/tex]

A second degree polynomial is one in which the highest degree of powers is of the second degree.

Basically, a polynomial in which the highest power is 2.

By observing the given polynomial, it is a second degree polynomial.

So, the answer is True.

Which ordered pair must be a solution in the graph of the linear inequalitybelow?(-2,2)(0, -2)

Answers

SOLUTION:

We want to determine which ordered pair must be a solution in the graph of the linear inequality. The point picked must be in the shaded region to be a solution.

Going through the options, we see that, the only ordered pair that is a solution there is;

[tex](-5,1)[/tex]

A woman invests $6300 in an account that pays 6% interest per year, compounded continuously.(a) What is the amount after 2 years? (Round your answer to the nearest cent.)$ (b) How long will it take for the amount to be $8000? (Round your answer to two decimal places.) yr

Answers

Given: A woman invests $6300 in an account that pays 6% interest per year, compounded continuously.

Required: a) To determine the amount after 2 years.

b) To determine how long it will take for the amount to be $8000.

Explanation: The amount, A after t years with an interest rate of r is given by-

[tex]A=Pe^{rt}[/tex]

Here,

[tex]\begin{gathered} P=6300 \\ r=\frac{6}{100} \\ =0.06 \\ t=2 \end{gathered}[/tex]

Substituting the values, we get-

[tex]\begin{gathered} A=6300e^{0.06\times2} \\ =7103.23 \end{gathered}[/tex]

Hence the amount after 2 years is $7103.23

Next, let t be the time it takes for the amount to be $8000-

[tex]\begin{gathered} 8000=6300e^{0.06t} \\ \frac{8000}{6300}=e^{0.06t} \\ \ln(1.2698)=0.06t \end{gathered}[/tex]

Further solving for t as-

[tex]t=3.98\text{ years}[/tex]

Hence, it takes 3.98 years for the amount to be $8000.

Final Answer: a) $7103.23

b) 3.98 years

Which of the following could be the end behavior of f(x) = -x6 + 3x4 + 8x3 – 4x2 – 6? f(x) → ∞ as x → ±∞f(x) → -∞ as x → ±∞f(x) → -∞ as x → -∞ and f(x) → ∞ as x → ∞f(x) → ∞ as x → -∞ and f(x) → -∞ as x → ∞

Answers

The given function f is given by:

[tex]f(x)=-x^6+3x^4+8x^3-4x^2-6[/tex]

Therefore, as:

[tex]x\to\infty,f(x)\to-\infty[/tex]

and

[tex]\begin{gathered} \text{ as } \\ x\to-\infty,f(x)\to-\infty \end{gathered}[/tex]

Therefore, the correct answer is:

f(x) → -∞ as x → ±∞

helpppppppppppppppp plssssssssss
The population of Orange County is represented by the function f(x)=87,000(0.9)x, where x is the number of years since 2010.

The population of Greene County was 78,000 in 2010, and has decreased exponentially at a rate of 8% each year.

How do the populations of these counties compare in 2015?

Drag a value or word to the boxes to correctly complete the statements.



Put responses in the correct input to answer the question. Select a response, navigate to the desired input and insert the response. Responses can be selected and inserted using the space bar, enter key, left mouse button or touchpad. Responses can also be moved by dragging with a mouse.
In 2015, the population of Orange County was approximately Response area and the population of Greene County was approximately Response area. In 2015, Response area County was more populous.

Answers

The population of Greene County will be more than Orange County in 2015.

What is comparison?

Comparing numbers, in maths, is defined as a process or method in which one can determine whether a number is smaller, greater, or equal to another number according to their values.

Given that, The population of Orange County is represented by the function f(x) = 87,000(0.9)x, where x is the number of years since 2010.

The population of Greene County was 78,000 in 2010, and has decreased exponentially at a rate of 8% each year.

Orange County population in 2015 = 87,000(0.9)^5 = 51372

Greene County population in 2015 = 78000(1+0.08)^5 = 51408

Hence, The population of Greene County will be more than Orange County in 2015.

For more references on comparison, click;

https://brainly.com/question/14047951

#SPJ1

Аis a solid consisting of two polygons which are parallel to each otherand all points between them.O A. cubeO B. prismO C. pyramidO D. triangle

Answers

The prism is a type of polyhedron formed by two parallel faces that are identical polygons called bases. These figures are joined by the lateral faces that are parallelograms.

According to the previous definition we can conclude that the answer is:

B. Prism

Make Sense and persevere An office manageris selecting a water delivery service. AcmeH2O charges a $15 fee and $7.50 per 5-gallonjug. Best Water charges a $24 fee and$6.00 per 5-gallon jug. How many 5-gallonjugs will the office have to buy each monthfor the cost of Best Water to be less than thatof Acme H20?

Answers

You have that Acme H2O charges 15 fee and7.50 per 5-gallon. Furthermore, Best water charges 24 fee and 6.00 per 5-gallon.

In order to find the amount of five-gallons that the office have to buy, you can write, in an akgebraic form, the previous realtions, just as follow:

24 + 6x < 15 + 7.5x

That is, cost of Best water less than cost of Acme H20. x is the amount of five-gallons

You solve the previous inequality, as follow:

24 + 6x < 15 + 7.5x subtract 6x both sides and subtract 15 both sides

24 - 15 + 6x - 6x < 15 - 15 + 7.5x - 6x simplify

9 < 1.5x divide between 1.5 both sides

9/1.5 < 1.5x/1.5 simplify

6 < x

6 < x is the same as x > 6. Hence, Office would have to buy lower than 6 five-gallons.

Stanley marked two points on the grid below to show the locations of the fiction section, point F, and the travel section, point T, in a bookstore.

Answers

EXPLANATION

We need to calculate the distance between the points (x₁,y₁)=(-8,-3) and (x₂,y₂)=(-3,8) applying the distance equation as shown as follows:

distance=

[tex]\text{Distance}=\sqrt[]{(x_2-x_1)^2+(y_2-y_1)^2}[/tex]

Substituting terms:

[tex]\text{Distance = }\sqrt[]{(-3-(-8))^2+(8-(-3))^2}[/tex]

Adding numbers:

[tex]\text{Distance}=\sqrt[]{(5)^2+(11)^2}=\sqrt[]{(25+121)}=\sqrt[]{146}[/tex]

The shortest distance is sqrt(146)

I need help with this problem if anyone can help me please do Find the value of the variable

Answers

Answer:

u=11

Explanation:

The statement "The quotient of 44 and 4 is u" can be represented mathematically as:

[tex]u=44\div4[/tex]

We can then solve the equation for u.

[tex]\begin{gathered} u=\frac{44}{4}=\frac{4\times11}{4} \\ \implies u=11 \end{gathered}[/tex]

The value of u is 11.

Help me please I don’t understand and I don’t get this

Answers

The solution:

Representing the given problem in a diagram, we have:

NEED HELP ASAPP!!!!!!

Answers

Answer:

second line

Step-by-step explanation:

no more than :

x≤ 52 (the dot is full, 52 is a value)

Which of the following options results in a graph that shows exponentialdecay?5 pointsO f(x) 0.4(0.2)^xf(x) = 4(4)^xf(x) = 0.7(1.98)^xOf(x) = 5(1+.1)^x

Answers

Answer

Option A is the answer.

f(x) = 0.4(0.2)ˣ

The value carrying the power of x is less than 1, so, this expression represents exponential decay.

Explanation

The key to knowing which expression is represents an exponential decay or exponential growth is the value of the number carrying the power of x.

If that number is greater than 1, then it represents exponential growth.

But, if that number is lesser than 1 (but greater than 0), then it represents exponential decay.

(2)ˣ represents exponential growth.

(0.5)ˣ represents exponetial decay.

f(x) = 0.4(0.2)ˣ

The value carrying the power of x is less than 1, so, this expression represents exponential decay.

f(x) = 4(4)ˣ

The value carrying the power of x is greater than 1, so, this expression represents exponential growth.

f(x) = 0.7(1.98)ˣ

The value carrying the power of x is greater than 1, so, this expression represents exponential growth.

f(x) = 5(1 + .1)ˣ

The value carrying the power of x is greater than 1, so, this expression represents exponential growth.

Hope this Helps!!!

What is the half-life of the goo in minutes? Find a formula for G(t), the amount of goo remaining at time t. How many grams of goo will remain after 68 minutes?

Answers

To solve this question on the half-life, we will use this expression:

[tex]\begin{gathered} G(t)=G_oe^{-kt} \\ \text{where G(t) is the remaining sample at time t.} \\ G_{o\text{ }}\text{ is the original sample} \\ K\text{ is a constant} \\ t\text{ is time} \end{gathered}[/tex]

To proceed in solving, we will need to find the value of constant k

[tex]\begin{gathered} G(t)=G_oe^{-kt} \\ G(t)=17.25 \\ G_o=276 \\ t=255 \\ \text{Now substitute the parameters above into the formula:} \\ 17.25=276e^{-k(255)} \\ \frac{17.25}{276}=e^{-k(255)} \end{gathered}[/tex][tex]\begin{gathered} 0.0625=e^{-k255} \\ \ln 0.0625=-255k \\ \frac{\ln 0.0625}{-255}=k \\ 0.0109=k \end{gathered}[/tex]

Now to get the half-life in minutes will be to get the time taken for the sample to go from 276g to 138g.

[tex]\begin{gathered} G(t)=G_oe^{-kt} \\ G(t)=138g \\ 138=276e^{-0.0109t} \\ \frac{138}{276}=e^{-0.0109t} \\ 0.5=e^{-0.0109t} \\ \ln 0.5=-0.0109t \\ \frac{\ln 0.5}{-0.0109}=t \\ 63.591\text{minutes = t} \end{gathered}[/tex]

The half-life is 63.59 minutes.

The formula for G(t) at time t is:

[tex]G(t)=276e^{-0.0109t}[/tex]

The amount of goo that will remain after 68 minutes is calculated using the formula above:

[tex]\begin{gathered} G(t)=276e^{-0.0109t} \\ t=68\text{ minutes} \\ G(t)=276e^{-0.0109(68)} \\ G(t)=276e^{-0.7412} \\ G(t)=131.5255\text{ grams} \\ G(t)\text{ = 131.53 grams (to 2 d.p)} \end{gathered}[/tex]

The amount of goo remaining after 68 minutes is 131.53 grams.

Rewrite the fraction (4/1)

Answers

Answer:

4

Explanation:

Given the fraction 4/1, this can be rewritten as 4 because any number divided by 1 can also be written as the number itself, both are equivalent.

10/13 ÷ 2 and 4/7.....

Answers

Given the expression;

[tex]\frac{10}{13}\div2\frac{4}{7}[/tex]

First we need to convert the mixed fraction 2 4/7 into improper fraction as shown;

[tex]\frac{10}{13}\div\frac{18}{7}[/tex]

Change the division sign to multiplication as shown;

[tex]\begin{gathered} =\frac{10}{13}\times\frac{7}{18} \\ =\frac{5}{13}\times\frac{7}{9} \\ =\text{ }\frac{35}{117} \end{gathered}[/tex]

Hence the answer to the expression is 35/117

Each expression represents the total number of dots in a pattern where n represents the step Select all the expressions that represent a quadratic relationship between the step number and the total number of dots. (If you get stuck, consider sketching the first few steps of each pattern as described by the expression.) A. I2 answer B. 2n C. non answer A D. nun E. n + 2 F.n=2 A &C • A, B, C B&C D. EF

Answers

We have the following:

We have that an option is quadratic when the same value is being multiplied twice, that is,

[tex]n\cdot n=n^2[/tex]

Therefore, among the answers the only quadratic options are A and C.

Add.(9s² - 3s) + (-6s - 9)

Answers

We have to add the expressions, grouping by similar terms:

[tex]\begin{gathered} \mleft(9s^2-3s\mright)+(-6s-9​) \\ 9s^2-3s-6s-9 \\ 9s^2-9s-9 \end{gathered}[/tex]

Answer: 9s²-9x-9

11. The table shows the distance, y, a car can travel in feet in x seconds. Speed or Car Time, x (seconds) 5 10 Distance, y (feet) 700 1,400 2,100 2,800 3,500 15 20 25 Based on the information in the table, which equation can be used to model the relationship between x and y? A. y = 140x B.y = 5x C. y = x + 140 D. y = x + 5

Answers

In this case, we'll have to carry out several steps to find the solution.

Step 01:

y = distance

x = time

equation = ?

Step 02:

y = k x

if y = 700ft , x = 5 seconds

700 = k * 5

700 /5 = k

140 = k

y = 140 x

The answer is:

y = 140 x

Jasper want to venture into the food stall. She plans to create a fund by making deposits of 2,000 in a bank that gives 4% interest compounded quarterly, how much money will be in the fund after 4 years?*228,892.31*101,891.58*228,805.31*103,891.58

Answers

We have to calculate the future value of making monthly deposits of $2000 in a bank that gives 4% interest compounded quarterly.

As the frequencies between the deposits and the compounding are different, we have to calculate a equivalent rate that compounds at the same frequency as the deposits (monthly) that keeps the same effective interest rate.

We have a nominal annual rate of 4% that compounds quarterly (m = 3). We can calculate the equivalent nominal annual rate as:

[tex]i=q\cdot\lbrack(1+\frac{r}{m})^{\frac{m}{q}}-1\rbrack[/tex]

where m = 3 is the current compounding subperiod, q = 12 is the new compounding subperiod and r = 0.04 is the current annual rate.

We replace the values and calculate:

[tex]\begin{gathered} i=12\cdot\lbrack(1+\frac{0.04}{3})^{\frac{3}{12}}-1\rbrack \\ i\approx12\cdot(1.0133^{\frac{1}{4}}-1) \\ i\approx12\cdot(1.003316795-1) \\ i\approx12\cdot0.003316795 \\ i\approx0.0398 \end{gathered}[/tex]

We can now use the interest rate i = 0.0398 compounded monthly as the equivalent rate.

We can calculate the future value of the annuity as:

[tex]FV=\frac{PMT}{\frac{i}{q}}\lbrack(1+\frac{i}{q})^{n\cdot q}-1\rbrack[/tex]

Where PMT = 2000, i = 0.0398, q = 12 and n = 4.

We can replace with the values and calculate:

[tex]\begin{gathered} FV=\frac{2000}{\frac{0.0398}{12}}\cdot\lbrack(1+\frac{0.0398}{12})^{4\cdot12}-1\rbrack \\ FV\approx603015.075\cdot\lbrack(1+0.003317)^{48}-1\rbrack \\ FV\approx603015.075\cdot\lbrack1.1722636-1\rbrack \\ FV\approx603015.075\cdot0.1722636 \\ FV\approx103877.55 \end{gathered}[/tex]

We get a future value of the annuity of $103,877.55.

We have some differences corresponding to the roundings made in the calculation, but this value correspond to the option $103,891.58.

Answer: $103,891.58

I paid $17.80 for 5 gallons of gas. using the unit rate. how much would you pay to fill all the way up if my car holds 14 gallons of gas ?

Answers

Answer: 67

Step-by-step explanation:


Stan stray kids. !!

what i believe… is that you find the unit rate( divide 17.8 by 5) then you would get 3.56 and then multiply 3.54 by 14 and you would get 49.84? Tell me if this is right or wrong please.

Find the number of permutations of the letters in the word. APPLICATION

Answers

SOLUTION:

We want to find the number of permutations of the letters in the word APPLICATION.

The word has 11 letters of which A,P,I are repeated two times.

Thus, the formula for the possible permutations of a word with repeated letters is;

[tex]=\frac{n!}{n_1!n_2!...n_k!}[/tex]

Thus, we have;

[tex]\frac{11!}{2!2!2!}=4989600[/tex]

A hotel swimming pool is made for semi circle and square. Find The perimeter of the swimming pool. Round your answer to the nearest tenth.

Answers

Combining the 4 semi-circles of the pool, we can make 2 circles, with a diameter of 10 yd.

The perimeter of a circle is calculated as follows:

[tex]P=\pi\cdot D[/tex]

where D is the diameter.

Then, the perimeter of the swimming pool is:

[tex]\begin{gathered} P=2\cdot\pi\cdot10 \\ P=62.8yd^{} \end{gathered}[/tex]

Select the conic section that represents the equation.4x2 - 25y2 = 100circleparabolaellipsehyperbola

Answers

We know that the equation of a circle is:

[tex](x-a)^2+(y-b)^2=r^2[/tex]

the equation a a parabola is:

[tex]y=ax^2+bx+c[/tex]

the equation

Enter the equation in standard form.y = 4x - 9

Answers

The general form of the standard line is:

[tex]Ax+By=C[/tex]

So, we need to change the given equation to the standard form

the given equation is;

[tex]y=4x-9[/tex]

Making x and y on the left side

So,

[tex]-4x+y=-9[/tex]

And can be written as:

[tex]4x-y=9[/tex]

Complete the table using the equation y = 7x +4. NO -1 0 1 2

Answers

We will have the following:

*x = -1 => y = 7(-1) + 4 = -3

*x = 0 => y = 7(0) + 4 = 4

*x = 1 => y = 7(1) + 4 = 11

*x = 2 => y = 7(2) + 4 = 18

*x = 3 => y = 7(3) + 4 = 25

Other Questions
ginny is raising pumpkins to enter a contest to see who can grow the heaviest pumpkin. her best pumpkin weighs 22 pounds and is growing at the rate of 2.5 pounds per week. martha planted her pumpkins late. her best pumpkin weighs 10 pounds but she expects it grow 4 pounds per week. define the "let" statements for x and y. then write equations that represent the weight of ginny and martha's pumpkins.Let x=Let y=ginny's equations=Martha's equation: (a) By how many percent is the torque of a motor decreased if its permanent magnets lose 3.9% of their strength? %(b) How many percent would the current need to be increased to return the torque to original values? % Represent the given condition using a single variable , x Please help us in figuring out this math problem so that we can move onto the next one thank you very much 2.3= p + 0.6What does p equal? Choose the right interjections and write in the blank spaces.a............ ! The poor man has died. (Alas, Pooh) !b............. I have passed (Oh, Hurrah) c...............! He is so cruel. (Alas, Pooh) d. ............ ! Well played. (Oh, Bravo)e............ ! Don't make a noise. (Oh, Hush) ! f.................How are you? (Hello, Oh)g. ..............! You live in a lovely place. (Aha, Alas) the Firgure shows two triangles on a coordinate grid: Which set of transformations have been performed on triangle ABC to form triangle A'B'C'? A) Dilation by a scale factor of 1/3 followed by reflection about the x-axisB) Dilation by a scale factor of 3 followed by reflection about the x-axis C) Dilation by a scale factor of 1/3 followed by reflection about the y-axis D) Dilation by a scale factor of 3 followed by reflection about the y-axis The information below shows the different genotypes that were observed during the meiosis simulation and the number of copies of each genotype that were present at the end of all rounds of meiosis. Use this info to answer the below question: EAD- 6EaD- 19Ead- 25eAD-25eAd- 19ead-6 Which genotype that were observed in one or more daughter cells was also present in the parental cell? Which were ONLY in the daughter cell? Which were ONLY in the parental cell? the drop down menu is the same on both sides What is a helicapterHi I keep getting a step wrong because Im getting a radio that isnt on here.. 30 points helps What are the coordinates of each vertex if the figure is rotated 180 clockwise about the origin?[G.CO.2, G.CO.4, G.C0.5] The following is a sample of 20 measurements.Answer b part Devonte creates a scatter plot of the relationship between his hourly pay in dollars, y, and the number of customers he serves at a coffee shop, X. He calculates the equation of the trend line to be y = 2.52 +7. Part A What does the y-intercept represent? Enter the correct answers in the boxes. per hour when he serves customers. The y-intercept represents that Devonte earns $ Which of the following is an Autotroph?A. TigerB. Primary consumerC. OaktreeD. Decomposer Will a truck that is 14 feet wide carrying a load that reaches 12 feet above the ground clear the semielliptical arch on the one-way road that passes under the bridge shown in the figure on the right? Error Analysis Denzel identified (3, 2) as a point on the line y - 2 = 2/3 (x + 3). What is the error that Denzel made? The vivandire sold viands and sandwiches. If she sold 300 total and 50 more viands than sandwiches, how many of each did she sell? You were helping your science teacher put away lab materials. They asked you to carry sand, iron filings, and salt in cups to put away. However, a fellow student left their backpack in the middle of the floor. You fell over it and dropped all of the materials. After sweeping up the mess, all of the sand, iron filings and salt are mixed together. Your job now is to use the physical properties of these substances to separate the mixture and return the pure substances to their separate containers.What physical property or properties might help you remove salt from the mixture? (2 points)What process will you use to separate the salt from the mixture? (2 points)What physical property of properties might help you remove iron from the mixture? (2 points)What process will you use to separate the iron from the mixture? (2 points)What physical property or properties might help you remove sand from the mixture? (2 points)What process will you use to remove sand from the mixture? (2 points)Now, list the steps that you will take to separate the three components of the mixture. Be specific enough that someone else could complete the mixture separation exactly as you have planned to do it. You may use bullet points and labeled diagrams to answer this question. (8 points, see rubric) What is the center and the radius of the circle: ( x + 7 ) 2 + ( y - 1 ) 2 = 9 ?